What is a cubic polynomial function in standard form with zeros 3 –2 and 2

I really need help with this. At least just a nudge.

Thanks

Answers

Answer 1

Answer:

x^3 - 3x^2 - 4x + 12

Step-by-step explanation:

Okay, so let's work backwards.

3, -2, 2 are the zeros, so we can use these numbers write the equation in factored form. (For right now, as a placeholder, I'll let y = 0)

(x - 3)(x + 2)(x - 2) = 0

If you want to check this, you can insert each of the zeros into the expression with a constant of the opposite sign.

So now, we can just expand the factored form. (I'll be doing this one equation at a time)

(x - 3)(x + 2) = x^2 + 2x - 3x - 6 = x^2 - x - 6

(x^2 - x - 6)(x - 2) = x^3 - 2x^2 - x^2 + 2x - 6x + 12 = x^3 - 3x^2 - 4x + 12

So,

The equation in standard form is x^3 - 3x^2 - 4x + 12

(Don't take my word for it though, now that I've told you how, feel free to do it on your own)


Related Questions

Estimate the cube root of 92​

Answers

Answer:

Cube root of 92 can be represented as 3√92. The value of cube root of one is 92.The nearest previous perfect cube is 64 and the nearest next perfect cube is 125.

Step-by-step explanation:

mathematics sat question

Answers

Answer: A) [tex]a^{2}[/tex] + a

add the two equations together to get A.

the ones cancel eachother out and you cant add [tex]a^{2}[/tex] to a

Answer:

A) a^2 + a

Step-by-step explanation:

So the problem says "sum" which means you have to add the expressions

(a^2) - 1 and a+1

Work:

((a^2) - 1) + (a + 1)

We can simplify only the constants

(a^2) + a + 1 - 1

(a^2) + a

so the answer is A

Ayuda please I need help me please

Answers

Answer:

sqrt(61)

Step-by-step explanation:

4-(-1) = 5

-3-(-9) = 6

sqrt(5^2+6^2) = sqrt(25+36) = sqrt(61)

Right answer gets brainlyest

Answers

Answer:

[tex]-\frac{7}{8}a-\frac{1}{6}[/tex] (the first box is 7/8 and the second box is 1/6)

Step-by-step explanation:

Step 1: Combine like terms.

[tex](-\frac{5}{8}a-\frac{1}{4}a)+(\frac{2}{3} -\frac{5}{6})[/tex] [tex](-\frac{5}{8}a -\frac{2}{8}a)+(\frac{4}{6}-\frac{5}{6})[/tex] [tex]-\frac{7}{8}a-\frac{1}{6}[/tex]

Therefore, the answer is [tex]-\frac{7}{8}a-\frac{1}{6}[/tex]. (Again, the first box is 7/8 and the second box is 1/6).

If student A is 160 cm tall, and the student B is 20% shorter than student A, what is the height of student B?

Answers

Answer:

128

Step-by-step explanation:

It’s actually quite simple all you have to do is find out what 20% of 160 is then subtract the 20% by 160 to get your final answer UwU

Lines 1 || m, then Zz=
a. 120 b. 60 c. 180 d. 90​

Answers

Answer:

bbbbbbbbbbbbbbbbbbbbb

What is the value of (4/5 a 1/3) x 5^2 + 5/6 expressed in simplest form

Answers

Answer:

The choice C.

[tex]12 \frac{1}{2} [/tex]

Step-by-step explanation:

[tex]( \frac{4}{5} - \frac{1}{3} ) \times {5}^{2} + \frac{5}{6} \\ \\ ( \frac{12}{15} - \frac{5}{15} ) \times 25 + \frac{5}{6} \\ \\ ( \frac{12 - 5}{15} ) \times 25 + \frac{5}{6} \\ \\ ( \frac{7}{15} ) \times 25 + \frac{5}{6} \\ \\ \frac{35}{3} + \frac{5}{6} \\ \\ \frac{70}{6} + \frac{5}{6} = \frac{75}{6} \\ \\ = \frac{25}{2} = 12 \frac{1}{2} [/tex]

I hope I helped you^_^

help me lolol skixisknwnd

Answers

Answer:

x=300

Step-by-step explanation:

First you had to find the scale factor of the image to do that you would have to divide 325 by 13 resulting in 25 then you would multiple 12 times 25 resuliting in 300.

212.65 x 10? = 21,265.0
A 102
B 109
C 103

Answers

Answer:

I'm confused what your asking because 212.65 x 10 does equal 21,265.0

3^3 log3 4
see image

Answers

=
51.529096 is your answer

True or False?
|-10| = -10

Answers

Answer:

false

Step-by-step explanation:

absolute value turns the number positive

I-10I=10

What is the LCM for 12 and 4

Answers

Answer:

12

Step-by-step explanation:

4 and 12 only share 12 which is the lowest number :D

(5 3/4- 3 1/6)× 4 4/5??????​

Answers

Answer:

12 2/5   or   12.4

Step-by-step explanation:

4t+10=20+2t (solve for t)

Answers

Answer:

t=5

Step-by-step explanation:

4t+10=20+2t

4t-2t=20-10

2t=10

t=10÷2

=5

1. Move 2t to the left-hand side and change its sign
4t + 10 - 2t = 20

2. Move 10 to the right-hand side and change its sign
4t - 2t = 20 - 10

3. Collect like terms
2t = 20 - 10

4. Subtract the numbers
2t = 10

5. Divide both sides by 2
2t = 10 ➗ 2

Write the value of t
t = 5

Last month, you joined an online strategy game. When you joined, there were 50 members. Since then, 41 members have quit. What is the percent change in the number of
members from last month to this month?
Amount Decrease

Answers

Answer:

82% decrease

Step-by-step explanation:

percent change = (new number - old number)/(old number) * 100%

There were 50 members.

41 left.

50 - 41 = 9

The new number of members is 9.

percent change = (9 - 50)/(50) * 100%

percent change = -82%

A negative percent change is a decrease.

Let A = {1,5, 17,21} and B =
{3,4,6,10). R SAXB, R = {(1,3).(5,6),(5,10),(21,6)}
What are 1, 5, and 21 related to? What is domain, codomain, and range of R?
a.

Answers

sa may picture yong answer

Step-by-step explanation:

sana makatulong

Solve.−4 2/5=x−1 1/4What is the solution to the equation?Enter your answer as a simplified mixed number in the box.x =

Answers

Answer:

[tex]\boxed{\sf x=-3\frac{3}{20}}[/tex]

Step-by-step explanation:

[tex]\sf -4\cfrac{2}{5}=x-1\cfrac{1}{4}[/tex]

Multiply Both sides by 20, Which is the LCM of 5, 4:

[tex]\mapsto \sf (4\times 5+2)=20x-5(4+1)[/tex]

Multiply 4 and 5= 20, and add 20 +2 =22

[tex]\mapsto \sf -4\times 22=20x-5(4+1)[/tex]

Multiply -4 * 22 = -88, and then add 4+1=5:

[tex]\mapsto \sf -88=20x-5\times 5[/tex]

Multiply -5 *5 = -25:

[tex]\mapsto \sf -88=20x-25[/tex]

Now, swap sides:

[tex]\mapsto \sf 20x-25=-88[/tex]

Add 25 to both sides:

[tex]\mapsto \sf 20x=-88+25[/tex]

[tex]\mapsto \sf 20x=-63[/tex]

Divide both sides by 20:

[tex]\mapsto \sf x=\cfrac{-63}{20}[/tex]

Now, convert an improper fraction to a mixed number: →

[tex]\mapsto \sf x= -3\cfrac{3}{20}[/tex]

_____________________________________

Find the sum
[tex]\sum_{k=1}^{\infty}\sum_{n=1}^{\infty} \frac{k}{ {2}^{n + k} } . \\ [/tex]
Correct answer will be mark as brainliest answer.​

Answers

[tex] \large\underline{\sf{Solution-}}[/tex]

Given series is

[tex]\rm \longmapsto\:\displaystyle\sum_{k=1}^{\infty}\rm \displaystyle\sum_{n=1}^{\infty}\rm \frac{k}{ {2}^{n + k} } [/tex]

can be further rewritten as

[tex]\rm \:  =  \: \:\displaystyle\sum_{k=1}^{\infty}\rm \displaystyle\sum_{n=1}^{\infty}\rm \frac{k}{ {2}^{k} . {2}^{n} } [/tex]

can be further rewritten as

[tex]\rm \:  =  \: \displaystyle\sum_{k=1}^{\infty}\rm \bigg[\dfrac{k}{ {2}^{k}}\displaystyle\sum_{n=1}^{\infty}\rm \dfrac{1}{ {2}^{n} } \bigg][/tex]

[tex]\rm \:  =  \: \displaystyle\sum_{k=1}^{\infty}\rm \dfrac{k}{ {2}^{k} }\bigg(\dfrac{1}{2} + \dfrac{1}{ {2}^{2} } + \dfrac{1}{ {2}^{3} } - - - \infty \bigg) [/tex]

So, its an infinite GP series with

[tex] \purple{\rm \longmapsto\:a = \dfrac{1}{2}}[/tex]

[tex] \purple{\rm \longmapsto\:r = \dfrac{1}{2}}[/tex]

We know,

Sum of infinite GP series with common ratio r ( - 1 < r < 1 ) and first term a is given by

[tex]\boxed{\tt{ \: \: S_ \infty = \frac{a}{1 - r} , \: \: provided \: that \: |r| < 1}}[/tex]

So, 1using this, we get

[tex]\rm \:  =  \: \displaystyle\sum_{k=1}^{\infty}\rm \frac{k}{ {2}^{k} }\bigg( \frac{ \frac{1}{2} }{1 - \frac{1}{2} }\bigg)[/tex]

[tex]\rm \:  =  \: \displaystyle\sum_{k=1}^{\infty}\rm \frac{k}{ {2}^{k} }\bigg( \frac{ \frac{1}{2} }{ \frac{2 - 1}{2} }\bigg)[/tex]

[tex]\rm \:  =  \: \displaystyle\sum_{k=1}^{\infty}\rm \frac{k}{ {2}^{k} }\bigg( \frac{ \frac{1}{2} }{ \frac{1}{2} }\bigg)[/tex]

[tex]\rm \:  =  \: \displaystyle\sum_{k=1}^{\infty}\rm \frac{k}{ {2}^{k} }[/tex]

[tex]\rm \:  =  \: \dfrac{1}{2} + \dfrac{2}{ {2}^{2} } + \dfrac{3}{ {2}^{3} } + \dfrac{4}{ {2}^{4} } + - - - \infty [/tex]

Let assume that

[tex]\rm \longmapsto\:S_ \infty   =  \: \dfrac{1}{2} + \dfrac{2}{ {2}^{2} } + \dfrac{3}{ {2}^{3} } + \dfrac{4}{ {2}^{4} } + - - - \infty [/tex]

Now, its an infinite Arithmetico Geometrico Series, So multiply by 1/2 we get

[tex]\rm \longmapsto\:\dfrac{1}{2} S_ \infty   =  \: \dfrac{1}{ {2}^{2} } + \dfrac{2}{ {2}^{3} } + \dfrac{3}{ {2}^{4} } + \dfrac{4}{ {2}^{5} } + - - - \infty [/tex]

On Subtracting above two equations, we get

[tex]\rm \longmapsto\:\dfrac{1}{2} S_ \infty   =  \: \dfrac{1}{2} + \dfrac{1}{ {2}^{2} } + \dfrac{1}{ {2}^{3} } + \dfrac{1}{ {2}^{4} } + - - - \infty [/tex]

[tex]\rm \longmapsto\:\dfrac{1}{2} S_ \infty   =  \: \dfrac{ \dfrac{1}{2} }{1 - \dfrac{1}{2} } [/tex]

[tex]\rm \longmapsto\:\dfrac{1}{2} S_ \infty   =  \: \dfrac{ \dfrac{1}{2} }{ \dfrac{1}{2} } [/tex]

[tex]\rm \longmapsto\:\dfrac{1}{2} S_ \infty   =  \: 1[/tex]

[tex]\bf\implies \:S_ \infty = 2[/tex]

Hence,

[tex] \\ \rm \longmapsto\:\boxed{\tt{ \: \: \: \: \displaystyle\sum_{k=1}^{\infty}\rm \displaystyle\sum_{n=1}^{\infty}\rm \frac{k}{ {2}^{n + k} } = 2 \: \: \: \: }} \\ [/tex]

10. Trevor rode his mountain bike 84 km in
3 h. Jillian rode 70 km in 2.5 h. Who is
the faster cyclist? How do you
know?

Answers

The volume of Trevor = 84 km+3h = 28 km The volume of Jillian = 70 km 2.5h = 28 km/ So they rode equally fast 28 = 28
Trevor:
84km / 3h = 28 km/h

Jillian:
70km / 2.5h = 28 km/h

They are going the same speed. We know this because speed=distance/time and they are both going 28km/h.

a
A shipping container is in the shape of a right rectangular prism with a length
of 13.5 feet, a width of 11 feet, and a height of 5-5 feet. The container is
completely filled with contents that weigh, on average, 0.93 pound per cubic
foot. What is the weight of the contents in the container, to the nearest
pound?

Answers

Answer:760 lbs

Step-by-step explanation:

The volume of a right rectanglar prism (i.e., a box) is length*width*height.

Volume = (13.5)*(11)*(5.5) = 817 ft^3

Density = 0.93 lb/ft^3

Weight of container contents = (817 ft^3)*(0.93 lg/ft^3) = 760 lbs

The answer to this question Is 170 lbs

Someone pls help me I will make you brain

Answers

Answer:

k=4

Step-by-step explanation:

Answer:

The answer is K=2 100% sure.

Which exponential functions have been simplified correctly? Check all that apply.

Answers

Answer:

A, C, and D have been simplified correctly

Match the percent on the left with the appropriate estimate on the right

32% 20%
22% 70%
74% 1/2
51% 30%​

Answers

Answer:

32% goes to 30%. 22% goes to 20%. 74% goes to 70%. 51% goes to 1/2.

Step-by-step explanation:

Just treat is as rounding. When you round, 32 goes to 30 because the 2 is 4 or less. 22 and 74 each respectively round to 20 and 70. 1/2 is equal to 50%. And 51% rounds to 50%, or 1/2.

The correct value of the percent on the left with the appropriate estimate on the right is,

32% - 30%

22% - 20%

74% - 70%

51% - 1/2

What is mean by Percentage?

A number or ratio that can be expressed as a fraction of 100 or a relative value indicating hundredth part of any quantity is called percentage.

To Calculate the percent of a number , divide the number by whole number and multiply by 100.

Given that;

To match the percent on the left with the appropriate estimate on the right.

Now, We can simplify as;

32% = 0.32

22% = 0.22

74% = 0.74

51% = 0.51

20% = 0.20

70% = 0.70

1/2 = 0.5

30% = 0.3

Thus, The correct value of the percent on the left with the appropriate estimate on the right is,

32% - 30%

22% - 20%

74% - 70%

51% - 1/2

Learn more about the percent visit:

https://brainly.com/question/24877689

#SPJ2

Finding unknown measures in similar triangles

Answers

Answer:

8

Step-by-step explanation:

8

The value of the missing side of the triangle is 24.

What is the similarity?

Similarity refers to the degree of resemblance or likeness between two or more things, concepts, or ideas. It can be measured in various ways, depending on the context and the characteristics being compared.

Scale factor refers to the ratio of the size of an object in the original form to its size in the scaled or transformed form. It is commonly used in geometry and mathematics to describe how the dimensions of an object have changed after a transformation such as scaling, stretching, or shrinking.

From the given figure the value of the missing number is calculated as,

36 / 24 = x / 16

x = ( 8 x 3 )

x = 24

Therefore, the value of an unknown number is x = 24.

To know more about similarities follow

https://brainly.com/question/25631497

#SPJ2

PLZ HELP I WILL Give Brainliest
Berto has $12 to put gas in his car. If gas costs $3.75 per gallon, which ordered pair relating number of gallons of gas, x, to the total cost of the gas, y, includes the greatest amount of gas Berto can buy?

Answers

Answer:

Is this a graph question?

Just divide 12 by 3.75, with that you get 3.2. So 3.2 (gallons) X 3.75 (price) = 13 so your points would be (3.2 and 12)

What are the first 10 numbers of pi?

Answers

The first 10 numbers of pi are 3.1415926535

[tex] \Large\begin{gathered} {\underline{\boxed{ \rm {\red{\pi \: = \: \frac{22}{7} }}}}}\end{gathered}[/tex]

[tex] \Large{{ { \blue{ \rm{{↬}}}}}} \rm \large \: \: \: {{ \Large{ \purple{ \rm{{ \frac{22}{7} \: = \: 3.1415926535 }}}}}} \: [/tex]

Mia is "helping" her mom pick up 27 toys that are strewn on the floor. Mia's mom puts five toys into the toy box every 40 seconds, but each time, immediately after those 40 seconds have elapsed, Mia takes three toys out of the box. How much time, in minutes, will it take Mia and her mom to put 27 toys into the box for the first time?

Answers

Answer:

8 min

Step-by-step explanation:fffffffffffffffffffffffffffffffffffffffffffffffffffffffffffffffffffffffffffffffffffffffffffffffffffffffffffffffffffffffffffffffffffffffffffffffffffffffffffffffffffffffffffffffffffffffffffffffffffffffffffffffffffffffffffffffffffffffffffffffffffffffffffffffffffffffffffffffffffffffffffffffffffffffffffffffffffffffffffffffffffffffffffffffffffffffffffffffffffffffffffffffffffffffffffffffffffffffffffffffffffffffffffffffffffffffffffffffffffffffffffffffffffffffffffffffffffffffffffffffffffffffffffffffffffffffffffffffffffffffffffffffffffffffffffffffffffffffffffffffffffffffffffffffffffffffffffffffffffffffffffffffffffffffffffffffffffffffffffffffffffffffffffffffffffffffffffffffffffffffffffffffffffffffffffffffffffffffffffffffffffffffffffffffffffffffffffffffffffffffffffffffffffffffffffffffffffffffffffffffffffffffffffffffffffffffffffffffffffffffffffffffffffffffffffffffffffffffffffffffffffffffffffffffffffffffffffffffffffffffffffffffffffffffffffffffffffffffffffffffffffffffffffffffffffffffffffffffffffffffffffffffffffffffffffffffffffffffffffffffffffffffffffffffffffffffffffffffffffffffffffffffffffffffffffffffffffffffffffffffffffffffffffffffffffffffffffffffffffffffffffffffffffffffffffffffffffffffffffffffffffffffffffffffffffffffffffffffffffffffffffffffffffffffffffffffffffffffffffffffffffffffffffffffffffffffffffffffffffffffffffffffffffffffffffffffffffffffffffffffffffffffffffffffffffffffffffffffffffffffffffffffffffffffffffffffffffffffffffffffffffffffffffffffffffffffffffffffffffffffffffffffffffffffffffffffffffffffffffffffffffffffffffffffffffffffffffffffffffffffffffffffffffffffffffffffffffffffffffffffffffffffffffffffffffffffffffffffffffffffffffffffffffffffffffffffffffffffffffffffffffffffffffffffffffffffffffffffffffffffffffffffffffffffffffffffffffffffffffffffffffffffffffffffffffffffffffffffffffffffffffffffffffffffffffffffff :)

Please Help ASAP !



.

Answers

Quinn's way takes a little more time because he didn't chose a good answer to multiply with 8 but gotten the answer anyways. Unlike Jewel, she had a answer that made things much quicker.

There are 3 teaspoons in 1 tablespoon. How many teaspoons are there in 6 tablespoons?

Answers

Answer:

18

Step-by-step explanation:

we can solve this as a ratio, from teaspoons to tablespoon is 3:1. if the tablespoon is 6 then the ratio becomes ?:6. Since we multiplied 1 by 6 we have to multiply 3 by 6 making the ratio 18:6.

PLS HELP ASAP FOR BRAINLIEST!!

Answers

Answer:

{x | 0 < x ≤ 5}

Step-by-step explanation:

The domain of a function describes all the values x could taken on in order to produce an output. You can notice the value of x is farthest left at a value of 0, which has an open dot and the value of x that is farthest right is at 5 with a closed dot. A closed dot includes the value while an open does not. This means x can be equal to 5, but not zero. We can write our domain as follows:

{x | 0 < x ≤ 5}

Other Questions
pls help i need this is connation a dictionary definition of a word? For a given function f(x) = x^2 - x + 1, the operation -f(x) = -(x^2 - x +1) will result in aA. horizontal shrinkB. reflection across the y-axisC. vertical shrinkD. reflection across the x-axis What is the answer I will give brainliest how long does it take for a body to become a skeleton The process of raising fish in a controlled area for the purpose of feeding the human population is called Pierre must learn three new piano pieces for his concert in two months. He is learning the first piece quickly, but the second piece is giving him a problem. He decides he needs to switch the order in which he practices, and he breaks his practices into shorter chunks of time. What skill is Pierre showing?AttitudeBeliefFlexibilitySelling Out Why did John Jay believe the Articles of Confederation were failing?O They kept people from having personal liberty.O They were unable to stop the lawlessness in Massachusetts.O They still allowed a king to be in control.o They unfairly increased the level of taxes on citizens. PLEASE HELP WILL GIVE BRAINLIEST AND 5 STARS AND A THANKS DUE IN 1 HOUR!~ LS2-1 The carrying capacity of a biological species in an environment is the maximum population size of thespecies that the environment can sustain indefinitely, given the food, habitat, water, and other necessitiesavailable in the environment. What factors would increase a carrying capacity for a population?I. habitat lossII. increase in food sourcesIII. access to clean waterIV. increase in competitionV. increase in diseasesII and IIIIII and IVII, IV, and VI, II, and V how do fatty acids travel through a cell membrane? 27 x 1000What is the answer and show me working out please? ASAP 40 pointsPicture attached your clients bust measurement is 32 inches. if you will draft the bodice pattern using the metric system, what is its equivalent to centimeters Evaluate the expression 90 = [(82 - 77).91 34. Which of the following is not a solution? a Salt dissolved in water b. Sand dissolved in water c. Sugar dissolved in water d. Oxygen dissolved in water What is the slope of the line that passes through the points (10,5) and (9,5)? Writeyour answer in simplest form. Evening Star by Edgar Allen Poe Which of an atom's electrons are involved in chemical reactions?a. The electrons farthest from the nucleus of the atomb. The electrons that make one isotope different from anotherc. The electrons that determine the atomic number of the atomd. The electrons closest to the nucleus of the atom code for apples and oranges codehs plz